What is the research hypothesis when using anova procedures?

Answers

Answer 1

When using ANOVA procedures, the research hypothesis is: there is no significance difference within the mean values of the groups.

What is a Research Hypothesis in ANOVA Procedure?

ANOVA procedure compares the mean values of different groups that are administered with treatments. The research hypothesis, such as the null hypothesis would be stated as: no significance difference in the mean values within the groups.

Thus, we can conclude that the research hypothesis when using the ANOVA procedures can be stated as a null hypothesis, which states that: there is no significance difference within the mean values of the groups.

Learn more about research hypothesis on:

https://brainly.com/question/20700422

#SPJ12


Related Questions

Which expression is the simplest form of 2x^3 - x^2 + 3 (x^3 - 4x^2)

Answers

The simplest form of the given expression is 5x^3-5x^2.

We have given that

[tex]2x^3 - x^2 + 3 (x^3 - 4x^2)[/tex]

We have to determine the simplest form of the given expression.

What is the distributive property?

The distributive property of binary operations generalizes the distributive law, which asserts that equality is always true in algebra. elementary.

Use the distributive property we get,

[tex]2x^3 - x^2 + 3 (x^3 - 4x^2)\\=2x^3 - x^2 +3x^3-12x^2\\[/tex]

Add like terms we get,

Therefore we get,

[tex]=5x^3-5x^2[/tex]

Therefore the simplest form of the given expression is 5x^3-5x^2.

To learn more about the  expression visit:

https://brainly.com/question/723406

#SPJ1

$Need help with this pls (quickest answer gets brainliest)

Answers

Answer:

-1/2

Step-by-step explanation:

IK

Which expression is equivalent to startfraction (4 p superscript negative 4 baseline q) superscript negative 2 baseline over 10 p q superscript negative 3 baseline endfraction? assume p not-equals 0, q not-equals 0.

Answers

The given expression is equivalent to [tex]\frac{p^{7}q}{160}[/tex]

What are indices?

An index is a small number that tells us how many times a term has been multiplied by itself.

The plural of index is indices.

Below is an example of a term written in index form :[tex]4^{3}[/tex]

4 is the base and 3 is the index.

We can read this as ‘4 to the power 3’

Another way of expressing [tex]4^{3}[/tex] is

4 x 4 x 4 = 64

Indices can be positive or negative numbers.

Given expression can be written as [tex]\frac{({4p^{-4}q})^{-2}}{10pq^{-3}}[/tex]

Now to simplify the given fractional expression :[tex]\frac{({4p^{-4}q})^{-2}}{10pq^{-3}}[/tex]

=   [tex]\frac{4^{-2}p^{8}q^{-2}}{10pq^{-3}}[/tex]                    By using the property of exponents is given by:

                                    [tex](a^{m})^{n}=a^{m n}[/tex]

=[tex]\frac{p^{7}q}{10 .16}[/tex]                              By using the property of exponents given by

                                       [tex]a^{m}a^{n}=a^{m + n}[/tex]  and  [tex]a^{-m}= \frac{1}{a^{m}}[/tex]

= [tex]\frac{p^{7}q}{160}[/tex]

Learn about indices here :

https://brainly.com/question/27327380

#SPJ4

Find the slope of every line that is parallel to line on the graph (0,-3) (5,-4)

Answers

The slope of the line parallel to the line on the graph (0,-3) (5,-4) is - 1 / 5

How to find the slope of a parallel lines?

Parallel lines have the same slope.

Therefore,

slope = m = y₂ - y₁ / x₂ - x₁

Therefore,

x₁ = 0

x₂ = 5

y₁ = -3

y₂ = -4

Therefore,

slope = -4 - (-3)  / 5 - 0

slope = -4 + 3 / 5

slope = - 1 / 5

learn more on slope here: https://brainly.com/question/8978282

#SPJ1

A passenger jet plane cruises at 550 knots. It enters a jet stream with
a tailwind speed of 120 knots. If the speed of sound is 661 knots,
will the jet travel faster or slower than sound during its journey?

Answers

Considering the direction of the wind, it is found that the jet travels faster than the sound during it's journey.

What is the ground speed?

The jet's speed, considering that there is a tailwind, is given by:

J = Plane speed + Wind speed.

In this problem, we have that the speeds are given as follows:

Plane: 550 knots.Wind: 120 knots.

Hence the jet's speed is given by:

J = 550 + 120 = 670 knots.

670 knots > 661 knots, hence the jet travels faster than the sound during it's journey.

A similar problem, involving plane and wind's speed, is given at brainly.com/question/25547425

#SPJ1

Which expression can be used to find the difference of the polynomials? (10m – 6) – (7m – 4) [10m (–7m)] [(–6) 4] (10m 7m) [(–6) (–4)] [(–10m) (–7m)] (6 4) [10m (–7m)] [6 (–4)]

Answers

The expression can be used to find the difference of the polynomials is [10m (–7m)] [(–6) 4]

Difference of polynomials

A polynomial is a function that has a leading degree of 3 and above.

Given the expression below

(10m – 6) – (7m – 4)

Expand

10m - 6 - 7m  + 4

Collect the like terms

10m - 7m - 6 + 4

Simplify

3m -2

Hence the expression can be used to find the difference of the polynomials is [10m (–7m)] [(–6) 4]

learn more on difference of polynomial here: https://brainly.com/question/2516456

#SPJ1

CAN SOMEONE HELP ME PLS?????

Answers

You are correct, P(red) = P(green) = 0.25.

John is 1/8 meter shorter than Paul, and Paul is 0.25 meter taller than Andrew. John's height is 13/4 meters, what is Andrews height?

Answers

Answer:

About 2.13

Step-by-step explanation:

13/4 multiply by 7/8 to get 91/32, then multiply by 0.75(3/4), which is 273/128

HELP MEEEEEEEEEEEEEEEEEEEEEEEEEEEEEEEEEEEEE
write and solve a proportion to complete the statement. Round to the nearest hundredth if necessary.
1. 6km ≈ ?mi
2. 2.5 L≈ ?gal
3. 90lb ≈ ?kg
pls help

Answers

Using proportion, the equivalent units of the following units rounded to the nearest hundredth are as follows:

6km =  3.73 miles2.5 Litres = 0.66 gallons90 pounds = 40.82 kg

How to use proportion to find equivalent units?

Using proportion,

1.60934 km = 1 miles

6 km = ?

cross multiply

distance(m) = 6 / 1.60934 = 3.7282364199 = 3.73 miles

1 litre = 0.264172 gallons

2.5 litres = ?

volume(gallons) = 2.5 × 0.264172 = 0.66 gallons

1 pounds = 0.453592 kg

90 pounds  = ?

cross multiply

weight(kg) = 40.82328 = 40.82 kg

learn more on proportion here; brainly.com/question/14063625

#SPJ1

Ahman has a lawn care business. he charges $25 per lawn to mow the grass. if his monthly expenses are $100, how many lawns must he mow in order to make a profit of at least $250 per month?

Answers

Answer:

14

Step-by-step explanation:

you can write an equation to represent the situation:

let x represent the number of lawns he has to mow

then, the monthly profit would be 25x - 100.

in order to make $250, this equation has to be equal to 250:

[tex]25x-100=250[/tex]

now, solve this:

add 100 to both sides

[tex]25x = 350[/tex]

divide both sides by 25

[tex]x = 14[/tex]

he must mow 14 lawns

Consider the systems of equations below. determine the number of real solutions for each system of equations. system a has real solutions. system b has real solutions. system c has real solutions.

Answers

System A has 2 real solutions, System B has 0 real solutions and System C has 1 real solution.

Given a system of equations for A is x²+y²=17 and y=-(1÷2)x, a system of equations for B is y=x²-7x10 and y=-6x+5 and a system of equations for C is y=-2x²+9 and 8x-y=-17.

For system A,

The two systems of equations are

x²+y²=17            ......(1)

y=-1÷2x             ......(2)

Substitute the value of equation (2) into equation (1) as

x²+(-x÷2)²=17

x²+(x²÷4)=17

Simplify the above equation by taking L.C.M. as

(4x²+x²)÷4=17

5x²=68

x²=68÷5

x=±3.688

Find the value of y by substituting the value of x in equation (2).

When x=3.688 then y is

y=-(1÷2)×3.688

y=-1.844

And When x=-3.688 then y is

y=-(1÷2)×(-3.688)

y=1.844

Thus, the points where the equations of system A intersect each other is (3.688,-1.844) and (-3.688,1.844)

So, the system of equations of A has 2 real solutions.

For system B,

The two systems of equations are

y=x²-7x+10            ......(3)

y=-6x+5                ......(4)

Substitute the value of equation (4) into equation (3) as

-6x+5=x²-7x+10

x²-7x+10+6x-5=0

x²-x+5=0

Simplify the above quadratic equation using the discriminant rule,

x=(-b±√(b²-4ac))÷(2a)

Here, a=1, b=-1 and c=5

Substitute the values in the discriminant rule as

x=(1±√(1-4\times 5\times 1))÷2

x=(1±√(-19))÷2

x=(1±√(19)i)÷2

Here, the value of x goes into the complex.

So, the system of equations of B has 0 real solutions.

For system C,

The two systems of equations are

y=-2x²+9            ......(5)

8x-y=-17            ......(6)

Substitute the value of equation (6) into equation (5) as

8x-(-2x²+9)=-17

8x+2x²-9+17=0

2x²+8x+8=0

Simplify the above quadratic equation using factorization method as

2x²+4x+4x+8=0

2x(x+2)+4(x+2)=0

(2x+4)(x+2)=0

x=-2,-2

Find the value of y by substituting the value of x in equation (5).

When x=-2 then y is

y=-2(-2)²+9

y=-8+9

y=1

Thus, the point where the equations of system C intersect each other is (-2,1)

So, the system of equations of C has 1 real solutions.

Hence, the system of equations for A is x²+y²=17 and y=-(1÷2)x having 2 real solution, a system of equations for B is y=x²-7x10 and y=-6x+5 having 0 real solution and a system of equations for C is y=-2x²+9 and 8x-y=-17 having 1 real solution.

Learn about system of equations from here brainly.com/question/12962074

#SPJ4

Mr. Lopez fruit salad recipe requires 3/4 of a cup of fresh peaches for 1 serving. He uses 9 cups of fresh peaches to prepare the salad. How many servings of the fruit salad did he prepare?

Answers

Answer:

6.75 servings

Step-by-step explanation:

[tex] \frac{3}{4} \times 9 = \frac{27}{4} = 6 \frac{3}{4} = 6.75[/tex]

Answer:

9 servings

Step-by-step explanation:

feel free to ask where you don't understand.

Find all solutions of the equation in the interval .
Write your answer in radians in terms of .
If there is more than one solution, separate them with commas.

Answers

The solutions to the trigonometric equation in the desired interval are given as follows:

[tex]\theta = \frac{\pi}{3}, \theta = \frac{5\pi}{3}[/tex]

What is the solution to the trigonometric equation?

The trigonometric equation is given by:

[tex]\sqrt{3}\cot{\theta} - 1 = 0[/tex]

Solving it similarly to an equation, we have that:

[tex]\sqrt{3}\cot{\theta} = 1[/tex]

[tex]\cot{\theta} = \frac{1}{\sqrt{3}}[/tex]

Since [tex]\cot{\theta} = \frac{1}{\tan{\theta}}[/tex], we have that the equation is equivalent to:

[tex]\tan{\theta} = \sqrt{3}[/tex]

The tangent is positive in the first and in the fourth quadrant. In the first quadrant, the angle [tex]\theta[/tex] with [tex]\tan{\theta} = \sqrt{3}[/tex] is:

[tex]\theta = \frac{\pi}{3}[/tex]

In the fourth quadrant, the equivalent angle is:

[tex]\theta = 2\pi - \frac{\pi}{3} = \frac{5\pi}{3}[/tex]

More can be learned about trigonometric equations at https://brainly.com/question/24680641

#SPJ1

Need Help Fast!!!!!! The graph of the piecewise function f(x) is shown. f(x) What is the range of f(x)?

Answers

Answer:

The second option

Step-by-step explanation:

If you look at the graph, it appears that from negative infinity to 0, the line is just constant, so the range of that would simply be the constant value or in this case 4. from 0 to infinity it appears the line is decreasing at a constant rate and should go towards negative infinity as x goes towards infinity. So the range would be -infinity < f(x) <= 4

8 1/2 - 3/8=?
pls answer fast ​

Answers

Answer:

65/8

Step-by-step explanation:

1) Turn all numbers into improper fractions: 17/2 - 3/8 = ?

2) Make all denominators the same number by finding the least common factor, which is 8. Multiply the denominator in 17/2, which is 2, by 4 to match the other denominator. And then multiply the numerator (17) by 4 as well so that the fraction still has the same value: 68/8 - 3/8 = 65/8

3) Can not simplify since there are no common factors between 65 and 8.

Andrea has 3 tiles. One is a regular octagon, one is a regular pentagon and one is a regular hexagon. Andrea thinks the 3 tiles will fit together perfectly, as shown in the diagram. Show calculations to prove that she is wrong.

Answers

From calculations, we can say that the given tiles will not fit together perfectly.

How to find the sum of interior angles of a Polygon?

If the tiles join perfectly at a point, sum of all angles around the joining point should be 360°.

Expression for the measure of the interior angle of a polygon,

Interior angle of a polygon = [(n - 2) * 180]/n

Interior angle of a pentagon = [(5 - 2) * 180]/5 = 108°

Interior angle of a hexagon = [(6 - 2) * 180]/6 = 120°

Interior angle of an octagon = [(8 - 2) * 180]/8 = 135°

To prove that the given tiles fit together perfectly → Sum of all the angles around the common point should be 360°

Sum of all interior angles = 108° + 120° + 135° = 363°

Therefore, given tiles will not fit together perfectly.

Read more about Interior angles of a Polygon at; https://brainly.com/question/224658

#SPJ1

The temperature fell at a rate of 0.65 °C/h. The temperature was recorded at 37 °C
at 6 p.m. Which function can be used to represent this situation?
Of(x) = 37 -0.65x
Of(x) = 0.65x - 37
Of(x) = 37x + 0.65
Of(x) = 0.65x+37

Answers

The linear function that can be used to represent the temperature in x hours after 6 pm is given by:

f(x) = 37 - 0.65x.

What is a linear function?

A linear function is modeled by:

y = mx + b

In which:

m is the slope, which is the rate of change, that is, by how much y changes when x changes by 1.b is the y-intercept, which is the value of y when x = 0, and can also be interpreted as the initial value of the function.

In this problem:

The y-intercept is the initial temperature of 37ºC.The slope is the rate of change of -0.65ºC/h.

Hence the function is:

f(x) = 37 - 0.65x.

More can be learned about linear functions at https://brainly.com/question/24808124

#SPJ1

its math help me out?

Answers

Its the first one

--------------------

Answer:

  $24 = $0.40(60)

Step-by-step explanation:

Match the input value and its location in the equation.

__

  $24 = $0.40(60)

_____

Additional comment

When input is liters and output is dollars, the constant of proportionality must have units of "dollars per liter." The dollar sign of these units is not shown in the left panel, but is shown on the answer choices. If you understand units conversion, this should not be a mystery. (The mystery is why the curriculum materials are inconsistent.)

Select all the functions that show an inverse variation...
a) y=6x
b) y=3/x
c) y=1/3x
d) y= -.07x
e) y= 2/x+4
f) xy=-3
PLEASE HELP ILL MARK BRAINLEST

Answers

A and d would be the answers for this question

Factor x² - 4x + 5.

A Prime
B (x + 5)(x - 1)
C (x - 5)(x - 1)
D (x + 5)(x + 1)

Answers

Answer:

[tex]\huge\boxed{\sf Option \ A}[/tex]

Step-by-step explanation:

Given expression:

= x² - 4x + 5

We can not factor out this expression, because of we apply mid-term break to it, the factors -4x will break into are -5x + x the coefficients of which when multiplied give -5 and not +5 (side term). This is a known rule in the mid-term break formula.

So, the given expression is a prime expression.

[tex]\rule[225]{225}{2}[/tex]

what does 5r-r-9=15 look like after combining like terms ?
and whats the answer for r?

Answers

The value of r from the expression is 6

Simplifying expression

Given the expression below;

5r-r-9=15

Add 9 to both sides

5r - r - 9 + 9 = 15 + 9

4r = 15 + 9

4r = 24

After combining the like terms the expression will be 4r = 24

4r/4 = 24/4

r = 6

Hence the value of r from the expression is 6

Learn more on expression here: https://brainly.com/question/723406

#SPJ1

Which expression has a value of 16 when n = 5?
StartFraction 25 Over n EndFraction + 7
30 minus 3 n
7 + StartFraction 45 Over n EndFraction
n cubed minus 114

Answers

An expression is defined as a set of numbers, variables, and mathematical operations. The correct option is C.

What is an Expression?

In mathematics, an expression is defined as a set of numbers, variables, and mathematical operations formed according to rules dependent on the context.

To solve the problem substitute the value of n as 5 in each expression and then simplify to check if it equals to 16 or not.

A.) (25/n) + 7

      = (25/5) + 7

      = 12

This is not the required expression.

B.) 30 - 3n

      = 30 - 3(5)

      = 15

This is not the required expression.

C.) 7+ (45/n)

       = 7 + (45/5)

       = 7 + 9

       = 16

This is the required expression.

D.) n² - 114

       = (5)³ - 114

       = 125 - 114

       = 11

This is not the required expression.

Hence, the correct option is C.

Learn more about Expression:

https://brainly.com/question/13947055

#SPJ1

sum of numbers 975,983,923,913 and 985 rounded upto hundredth place is

Answers

4,779 if you are just adding to find the sun of the numbers

Answer:

4800

Step-by-step explanation:

If a question is asking for the "sum" of numbers, this just means we have to add them altogether.

975+983+923+913+985 = 4779

To find the hundredth place, we use our place value. 4 is in the thousands column, and 7 is in the hundreds. Once we find the hundreds, we need the number to the right of it (7).

7 > 5 so we round up. 7 becomes 8.

4800.

Work out, giving your answer in its simplest form:
2/3 x 3 3/5

Answers

Answer: 12/5

Step-by-step explanation:

[tex]3 \frac{3}{5}=\frac{18}{5}\\\\\implies \frac{2}{3} \times 3 \frac{3}{5}=\frac{2}{3} \times \frac{18}{5}\\\\=\frac{36}{15}\\\\=\boxed{\frac{12}{5}}[/tex]

The weight of a cat is normally distributed with a mean of 9 pounds and a standard deviation of 2 pounds. Using the empirical rule, what is the probability that a cat will weigh less than 11 pounds?

Answers

If the value of the z-score is 1. Then the probability that a cat will weigh less than 11 pounds will be 0.84134.

What is the z-score?

The z-score is a statistical evaluation of a value's correlation to the mean of a collection of values, expressed in terms of standard deviation.

The z-score is given as

z = (x - μ) / σ

Where μ is the mean, σ is the standard deviation, and x is the sample.

The weight of a cat is normally distributed with a mean of 9 pounds and a standard deviation of 2 pounds.

Then the probability that a cat will weigh less than 11 pounds will be

The value of z-score will be

z = (11 – 9) / 2

z = 1

Then the probability will be

P(x < 11) = P(z < 1)

P(x < 11) = 0.84134

Thus, the probability that a cat will weigh less than 11 pounds will be 0.84134.

More about the z-score link is given below.

https://brainly.com/question/15016913

#SPJ1

(02.07)
The equation below shows the relationship between the
temperature in degrees Celsius, C, and degrees Fahrenheit, F:
H
C
(F-32)
Which of the following formulas correctly solves for F? (1 point)

Answers

Answer:

[tex]\boxed {\frac{F -32}{9} = \frac{C }{5}}[/tex]

Step-by-step explanation:

The correct relation between degrees Celsius, °C, and degrees Fahrenheit, °F is :

[tex]\boxed {\frac{F -32}{9} = \frac{C }{5}}[/tex]


[tex]\{ \frac { ( \sqrt { 3 } ) \times 3 ^ { - 2 } } { ( \sqrt { 5 } ) ^ { 2 } } \} ^ { \frac { 1 } { 2 } }[/tex]solve this equation

Answers

Answer:

Step-by-step explanation:

Exponent law:

    [tex]\sf \bf a^m * a^n = a^{m+n}\\\\ (a^m)^n = a^{m*n}[/tex]

    [tex]\sf a^{-m}=\dfrac{1}{a^m}[/tex]

       First convert radical form to exponent form and then apply exponent law.

 [tex]\sf \sqrt{3}=3^{\frac{1}{2}}\\\\\sqrt{5}=5^{\frac{1}{2}}[/tex]

[tex]\sf \left(\dfrac{(\sqrt{3}*3^{-2}}{(\sqrt{5})^2}\right)^{\frac{1}{2}}= \left(\dfrac{3^{\frac{1}{2}}*3^{-2}}{(5^{\frac{1}{2}})^2} \right )^{\frac{1}{2}}[/tex]

                      [tex]= \left(\dfrac{3^{\frac{1}{2}-2}}{5^{\frac{1}{2}*2}}\right)^{\frac{1}{2}}\\\\=\left(\dfrac{3^{\frac{1-4}{2}}}{5}\right)^{\frac{1}{2}}\\\\=\left(\dfrac{3^{\frac{-3}{2}}}{5}\right)^{\frac{1}{2}}\\\\=\dfrac{3^{\frac{-3}{2}*{\frac{1}{2}}}}{5^{\frac{1}{2}}}\\\\ =\dfrac{3^{{\frac{-3}{4}}}}{5^{\frac{1}{2}}}[/tex]

Each of 36 students at a school play bought either a cup of orange juice or a sandwich. A cup of orange juice costs $1 and a sandwich costs $3. The total amount collected was $76. How many students bought orange juice, and how many bought a sandwich?

Let represent the number of students who bought a cup of orange juice and represent the number of students who bought a sandwich. Then the problem can be represented by this system of equations:

+ 3 = 76
+ = 36

Answer the questions to solve the problem.

1. Explain what you should do with the two equations to eliminate one of the variables.

Answers

In order to eliminate one of the variables, subtract one of the equation from the other equation.

How to eliminate one of the variables?

Given these equations:

o + 3s = 76 equation 1

o + s = 36 equation 2

Where:

o = number of orange juice bought

s = number of sandwiches bought

In order to eliminate one of the variables, subtract equation 2 from equation 1. The result is :

2s = 40

s = 20

To learn more about simultaneous equations, please check: https://brainly.com/question/25875552

#SPJ1

Inventory was taken on Day 1, Day 2, Day 5, Day 6, and Day 7. Norris wants to know roughly what the inventory was on Day 3.

What should he do to estimate the inventory on Day 3?


He should use the value for Day 2 because it is closest in value to Day 3.


He should randomly pick a point near the fitted line where x=3.


He should evaluate the function f(x)=−1/3 x+4 for y=3.


He should evaluate the function f(x)=−1/3 x+4 for x=3.

Answers

Answer:

A

Step-by-step explanation:

guessing

He should evaluate the function f(x)=−1/3 x+4 for x=3  , Option D is the correct answer.

What in Interpolation ?

Interpolation is when the line of best fit is used to determine the value of a point that is within the range of plotted points.

It is given to find the value at Day 3 which lies in the range of the points used for plotting

The line for best fit is

f(x) = (-1/3)x +4

At x = 3

f(3) = (-1/3) * 3 +4

f(3) = -1+4 = 3

Therefore He should evaluate the function f(x)=−1/3 x+4 for x=3

Option D is the correct answer.

To know more about Interpolation

https://brainly.com/question/18768845

#SPJ1

answer?.............

Answers

Answer:

-21

Step-by-step explanation:

My guess would be -21 Im not sure but that is my guess because x=5 the normal answer would be negative 25 so my guess would negative 21
Other Questions
5. The nurse is monitoring a client receiving 0.9% Sodium Chloride, 1 liter, running at 75mL/hour. How long will it take for these fluids to infuse? Round the answer to the whole hour. What is a hypothesis?A. an educated explanation for the set of observationsB. an organized way of recording dataC. the results of an experimentUdens What is the standard form of this function? f(x) = (x-4) + 2 m [39/(15-2)|-(2*9)someone solves this please, I keep getting different answers and everytime I look up the answer every website shows a different answer.. Productivity is the Blank______ at which goods and services are produced based upon total output given total inputs. Multiple choice question. rate input output process Select the correct answer.Jamle's research question is: "How are substances found on the Moon different from those found on the Earth?"Which quotation from the source provides the best Information to address this research question?OA "Moon dust isn't like the stuff that collects on a bookshelf or on tables - It's ubiquitous and abrasive, and itclings to everything."OB. Lunar dust] clogged the camera equipment and scratched helmet visors so badly that astronauts haddifficulty seeing."OC. "Here on Earth, particulate matter is a form of air pollution generated by forest fires, volcanic eruptions,and burning fossil fuels."OD. "The Canary-S... can measure a variety of pollutants, including particulate matter, carbon monoxide,methane, sulfur dioxide, and volatile organic compounds."LResetNextSign out6:22 what is the sum of first 6 terms Is 9.3 X 100^9 in scientific notation M1 = 3M M2 = 1.25M V1 = V1=.125L unkown What is V2? According to this excerpt, how has Odysseus changed over the course of his adventure? What should you substitute for y in the second equation (bottom equation) in order to solve the system by the substitution method? b) Another sequence is defined using this term-to-term rule, Un+1 = kun + r where k and rare constants. Given that u = 37, U= 188 and u3 = 943, find the value of k and the value of r. b ) Another sequence is defined using this term - to - term rule , Un + 1 = kun + r where k and rare constants . Given that u = 37 , U = 188 and u3 = 943 , find the value of k and the value of r . Angies rotation maps triangle XYZ to triangle XYZ. Which describes the rotation? Select the correct answer.If the graphs of the linear equations in a system are parallel, what does that mean about the possible solution(s) of the system?OA. The lines in a system cannot be parallel.There are infinitely many solutions.B.OC. There is exactly one solution.D. There is no solution.ResetNext In this excerpt from H. H. Munros The Open Window, which line reveals the effect that Framtons conversation has on other characters?She rattled on cheerfully about the shooting and the scarcity of birds, and the prospects for duck in the winter. To Framton it was all purely horrible. He made a desperate but only partially successful effort to turn the talk on to a less ghastly topic; he was conscious that his hostess was giving him only a fragment of her attention, and her eyes were constantly straying past him to the open window and the lawn beyond. It was certainly an unfortunate coincidence that he should have paid his visit on this tragic anniversary.The doctors agree in ordering me complete rest, an absence of mental excitement, and avoidance of anything in the nature of violent physical exercise, announced Framton, who laboured under the tolerably widespread delusion that total strangers and chance acquaintances are hungry for the least detail of ones ailments and infirmities, their cause and cure. On the matter of diet they are not so much in agreement, he continued.No? said Mrs. Sappleton, in a voice which only replaced a yawn at the last moment. Then she suddenly brightened into alert attentionbut not to what Framton was saying.Here they are at last! she cried. Just in time for tea, and dont they look as if they were muddy up to the eyes!A He made a desperate but only partially successful effort to turn the talk on to a less ghastly topic;B The doctors agree in ordering me complete rest, an absence of mental excitement, and avoidance of anything in the nature of violent physical exercise, C On the matter of diet they are not so much in agreement, he continued.D No? said Mrs. Sappleton, in a voice which only replaced a yawn at the last moment.I NEED SOME HELP (: How is democracy in the United States today different from democracy in ancient Athens?A) In the United States, "democracy" means "rule by the people."B) In the United States, all citizens can vote.C) In the United States, all citizens can present their opinions to the government.D) In the United States, citizens must elect representatives to speak for them in the government. ASAPPP WILL GIVE BRAINLEST!! Which of the following is TRUE about the economic system of Mercantilism? A.)It promoted government regulation of trade over all lands and colonies under a nation's control . B.)It required poor people to work for others in the New World for a number of years in exchange for the cost of their travel from Europe. C.)It only applied to merchants and artisans. D.) it was a system based on the theory that Earth's resources are limited . E.) It was the economic system of European Empires in the 16th-18th century . What is the centerpiece of new mexicos international ufo museum?. Suppose we have already saved $55 towards the cost of a new television set.We plan to save $6 more each week for the next several weeks.i) Write an equation for the total amount T we will have after w weeks from now.ii) Find the total amount that would be saved after 8 weeks.